Need help ASAP
I will mark Brainleist

Need help part A, B and C

Roller Coaster Crew
Ray and Kelsey have summer internships at an engineering firm. As part of their internship, they get to assist in the planning of a brand new roller coaster. For this assignment, you help Ray and Kelsey as they tackle the math behind some simple curves in the coaster's track.

Part A
The first part of Ray and Kelsey's roller coaster is a curved pattern that can be represented by a polynomial function.
1. Ray and Kelsey are working to graph a third-degree polynomial function that represents the first pattern in the coaster plan. Ray says the third-degree polynomial has four intercepts. Kelsey argues the function can have as many as three zeros only. Is there a way for the both of them to be correct? Explain your answer.

2. Kelsey has a list of possible functions. Pick one of the g(x) functions below and then describe to Kelsey the key features of g(x), including the end behavior, y-intercept, and zeros.
g(x) = (x + 2)(x − 1)(x − 2)
g(x) = (x + 3)(x + 2)(x − 3)
g(x) = (x + 2)(x − 2)(x − 3)
g(x) = (x + 5)(x + 2)(x − 5)
g(x) = (x + 7)(x + 1)(x − 1)

3. Create a graph of the polynomial function you selected from Question 2.

Part B
The second part of the new coaster is a parabola.

4. Ray needs help creating the second part of the coaster. Create a unique parabola in the pattern f(x) = (x − a)(x − b). Describe the direction of the parabola and determine the y-intercept and zeros.

5. Create a graph of the polynomial function you created in Question 4.

Part C
6. Now that the curve pieces are determined, use those pieces as sections of a complete coaster. By hand or by using a drawing program, sketch a design of Ray and Kelsey's coaster that includes the shape of the g(x) and f(x) functions that you chose in the Parts A and B. You do not have to include the coordinate plane. You may arrange the functions in any order you choose, but label each section of the graph with the corresponding function for your instructor to view.

Answers

Answer 1

Answer:

Part A

1. A third degree polynomial has exactly three zeros. This is because its highest power (or degree) is 3. Ray would only be correct in saying that a polynomial can have 4 zeros if it is a 4th degree polynomial. Since in this case, a 4rd degree polynomial is being discussed, Kelsey is correct.

2. Function chosen: g(x) = (x + 2)(x - 1)(x - 2)

End behaviour: x —> ∞, g(x) —> ∞

                         x —> - ∞, g(x) —> -∞

Zeros/x intercepts of the function: -2, 1, 2

y intercept: g(x) = (x + 2)(x - 1)(x - 2)

                   g(x) = (0 + 2)(0 - 1)(0 - 2)

                   g(x) = (2)(-1)(-2)

                   g(x) = 4

3. See Graph 1

Part B

4. f(x) = (x + 5)(x + 3)

Direction: opens upward

Zeros/x intercepts: -5, -3

y intercept: 15

5. See graph 2

Part C

You can draw the roller coaster yourself using the graphs below

Need Help ASAPI Will Mark BrainleistNeed Help Part A, B And C Roller Coaster CrewRay And Kelsey Have
Need Help ASAPI Will Mark BrainleistNeed Help Part A, B And C Roller Coaster CrewRay And Kelsey Have

Related Questions

If m<1=72 then find the value of m<2

A.18
B.108
C.72
D.144

Answers

U can use both of ways to solve this question

How many triangles are created in the following situation:
m/A=35°, a = 12,b=16
1 triangle
2 triangles
3 triangles
no triangle

Answers

Answer:

Step-by-step explanation:

3 things of triangle are given.

2 sides + 1 angle

so, possible triangles = 1 ans

Calculate the area of a rectangular field with perimeter 88 cm and length 18 cm​

Answers

The area of the rectangular field is 468cm².

Step-by-step explanation:

If, length is 18cm; width is unknown( let x represent width). Then,

Perimeter of a rectangle= Length + Width+ Length + Width

88= 18 + x + 18 + x

88= 36 + 2x

88-36 =2x

52= 2x

52/2=2x/2

26=x

•x=26cm

The width of the rectangle is 26cm.

Area of a rectangle= Length × Width

=18cm×26cm

= 468cm².


[tex] \sqrt{7} [/tex]
multiplicative inverse (reciprocal)​

Answers

Answer:  [tex]\frac{\sqrt{7}}{7}[/tex]

========================================================

Explanation:

The reciprocal of x is 1/x where x is nonzero. Multiplying x with 1/x leads to 1. For example, the numbers 9 and 1/9 are multiplicative inverses of each other.

We'll stick 1 over the given square root expression. Then follow these steps to rationalize the denominator.

[tex]\frac{1}{\sqrt{7}}\\\\\\\frac{1*\sqrt{7}}{\sqrt{7}*\sqrt{7}}\\\\\\\frac{\sqrt{7}}{(\sqrt{7})^2}\\\\\\\frac{\sqrt{7}}{7}\\\\[/tex]

In short, [tex]\frac{1}{\sqrt{7}}=\frac{\sqrt{7}}{7}[/tex]

The diameter of an electric cable is normally distributed, with a mean of 0.9 inch and a standard deviation of 0.02 inch. What is the probability that the diameter will exceed 0.92 inch? (You may need to use the standard normal distribution table. Round your answer to three decimal places.)

Answers

The probability that the diameter of the electric cable that is normally distributed will exceed 0.92 inch is 0.841

What is probability?

Probabilities are used to determine the chances, likelihood, possibilities of an event or collection of events

How to determine the probability?

The given parameters are:

Mean = 0.9

Standard deviation = 0.2

Calculate the z-score at x = 0.92 using

[tex]z = \frac{x - \mu}{\sigma}[/tex]

This gives

z = (0.92 - 0.9)/0.02

Evaluate the numerator; subtract 0.9 from 0.92

z = 0.02/0.02

Divide 0.02 by 0.02. This gives

z = 1

The probability is then represented as:

P(x > 0.92) = P(z > 1)

Next, we look up the value of the z table of probabilities

From the z table of probabilities, we have:

P(x > 0.92) = 0.841

Hence, the probability that the diameter of the electric cable that is normally distributed will exceed 0.92 inch is 0.841

Read more about probability at:

https://brainly.com/question/11234923

#SPJ1

10. What is the product of 6% and 14%

Answers

the answer is 0.0084

The day you were born your Grandparents started to save money for your future education. They decided to deposit $3,000 at the end of each year, for eighteen years, in a savings account that pays 7.5% per year, compounded annually.

On your 19th birthday you get admission into university for a four-year degree course. You calculate that you require approximately $35,000, for your fees, books and living expenses for each year. Assuming you keep the money in the bank accruing the same interest, and withdraw $ 35,000, at the beginning of each year.

a) Calculate the total value of this investment at the end of the term?

b) Determine the total interest earned?

Will your inheritance last you for the full tenure of your four years at the university?

Answers

The total value of the investment is $107,032.16.

The total interest earned is $53,032.16

The inheritance would last the tenure of the university.

What is the total value of the investment?

The formula that can be used to determine the future value of the 18-year annuity is: annuity factor x amount deposited yearly

Annuity factor = {[(1+r)^n] - 1} / r

Where:

r = interest rate n = number of years

$3000 x [(1.075^18 - 1) / 0.075] = $107,032.16

Amount deposited in the course of 18 years = (3000 x 18) = 54,000

Interest earned = 107,032.16 - 54,000 = $53,032.16

To learn more about annuities, please check: https://brainly.com/question/24108530

#SPJ1

a infant grew 2/4 inches in the first month 7/4 inches in the third month . first find the total inches the infant grew over the three months . then find the difference in the infants from the second month to the third month

Answers

The fraction computed shows that the total inches the infant grew over the three months is 3 3/4 inches.

How to compute the fraction?

First month = 2/4 inches

Second month = 1 inch

Third month = 7/4 inches.

The total inches the infant grew over the three months will be;

= 2/4 + 1 + 7/4

= 3 1/4 inches.

The difference in the infants from the second month to the third month will be:

= 7/4 - 1

= 3/4 inches.

Learn more about fractions on:

brainly.com/question/78672

#SPJ1

Find the domain of the function. Express the exact answer using interval notation.

f(x) = 2/3x+8

Answers

Answer:

(-oo, -8/3) U (-8/3, oo)

Step-by-step explanation:

Hope this helps.

The domain of the equation f(x) = 2/(3x + 1) is (-∝, -8/3) U (-8/3, ∝)

How to calculate the domain of the equation?

From the question, we have the following parameters that can be used in our computation:

f(x) = 2/(3x + 1)

The above equation is a rational function

The rule of a function is that

The domain is the set of all input numbers

For the domain, we set the denominator not equal to 0

So, we have

3x + 8 ≠ 0

Evaluate

x ≠ -8/3

When represented as interval notation, we have

Domain = (-∝, -8/3) U (-8/3, ∝)

Hence, the domain is (-∝, -8/3) U (-8/3, ∝)

Read more about domain at

brainly.com/question/27910766

#SPJ2

a vendor is thinking about changing the number of sweatshirts he brings to tan event. to make sure he doesn't run out, he plans to bring more of the size most likely to be sold. the table shows the number of sweatshirts of each size sold at the vendors last event and the number he had for sale. which size should he bring more of?

Answers

The sweatshirt size which the vendor should bring more of by virtue of that which is most likely to be sold is; Choice A; Extra-large.

Which sweatshirt size should the vendor bring more of?

If follows from the task content that the

ratio of extralarge shirts sold relative to the quantity he had to sell is

= 99/108

= 11/12,

This is a ratio which is larger than any other shirt.

Consequently, the vendor must bring more of the X-large shirts to avoid running out of them.

Read more on ratios;

https://brainly.com/question/13513438

#SPJ1

This is the photo for the last question

Answers

All the angles required to complete each sentence are:

m ∠ 4 = 60, m ∠ 2 = 120m ∠ 3 = 110, m ∠ 4 = 70m ∠ 2 = x, m ∠ 1 = 180 - x

How to find measures of missing angles by Euclidean geometry

In this question we must take advantage of definitions and theorems of Euclidean geometry to complete the three sentences seen in the figure. Now we proceed to present each sentence completed in detail:

If m ∠ 5 = 60, then m ∠ 4 = 60 by alternal internal angles between parallel lines and m ∠ 2 = 120 by supplementary angles.If m ∠ 7 = 110, then m ∠ 3 = 110 by corresponding angles between parallel lines and m ∠ 4 = 70 by supplementary angles. If m ∠ 6 = x, then m ∠ 2 = x by corresponding angles and m ∠ 1 = 180 - x by supplementary angles.

To learn more on angles: https://brainly.com/question/7116550

#SPJ1

Assume that the random variable X is normally distributed, with mean μ = 90 and standard deviation σ = 12. Compute the probability P(X < 105).

Answers

The probability P(X < 105), assuming that the random variable X is normally distributed, with mean μ = 90 and standard deviation σ = 12 is calculated to be 0.8944.

When a random variable X is normally distributed, with the mean as μ, and standard deviation as σ, then the probability P(X < x) is calculated using the Z-score table, as the probability P(Z < z), where z is calculated using the formula, z = (x - μ)/σ.

In the question, we are asked to find the probability P(X < 105), assuming that the random variable X is normally distributed, with mean μ = 90 and standard deviation σ = 12.

We calculate as follows:

P(X < 105)

= P(Z < (105-90)/12)

= P(Z < 1.25)

Now we check the z-score table for the value of z as 1.25.

On the left column we choose the value 1.2.

On the top row, we choose the value .05.

The intersection of these, gives us the value,

P(Z < 1.25) = 0.8944.

Thus, the probability P(X < 105), assuming that the random variable X is normally distributed, with mean μ = 90 and standard deviation σ = 12 is calculated to be 0.8944.

Learn more about probability for normal distribution at

https://brainly.com/question/14530400

#SPJ1

Help Me Please with this Math Money Question It Wont Me Screenshot But




Question: CD's are 5 for $30.00.

What is the price of 3 CD's?



Answers:
A) $18.00


B) $15.00


C) $6.00


D) $3.00

Answers

Answer:

A

Step-by-step explanation:

$30.00 ÷ 5 = $6.00

$6.00 × 3 = $18.00

Answer 18 (a)
First we need to find out the cost of one CD
We can do that by 30/5 =6
One CD cost 6 $
But the questions ask the cost of 3 CD
So the price for 3 CD is 18

a two digit number has twice as many ones as tens twice the original number is 9 more than the reversed number find the original number

Answers

Answer: 36

Step-by-step explanation:

As this is a two-digit number, it will have a digit in its 10's place and a digit in its 1's place. We can represent the 10's digit using x and the 1's digit using y.

Since the ones is twice the number of tens, this means that y is equal to 2*x. Let's put this into an equation.

[tex]y=2x[/tex]

If the 10's digit is x and the 1's digit is y, the original number would be [tex]10x + y[/tex], as x must be multiplied by 10 to get it to the 10's place. Similarly, the reversed number would be [tex]10y + x[/tex], as y must be multiplied by 10 to get it to the 10's place.

Twice the original number (10x+y) is equal to 9 more than (i.e. 9 plus) the reversed number. We can put this into an equation to help us answer the question.

[tex]2(10x+y)=9+10y+x\\20x+2y=9+10y+x\\19x+2y=9+10y\\19x=9+8y[/tex]

Now we have a system of two equations.

[tex]y=2x\\19x=9+8y[/tex]

We can solve this system by substituting y for 2x in the second equation. Then, we can isolate and get the value of x.

[tex]19x=9+8(2x)\\19x=9+16x\\3x=9\\x=3[/tex]

Now that we have the value of x, let's put it back into the first equation and solve for y.

[tex]y=2(3)\\y=6[/tex]

Remember that x is the 10's digit and y is the one's digit of our answer. Since x is 3 and y is 6, our answer is 36.

Checking

We can quickly check if our answer is right by making sure both conditions in the question are met.

6 (the one's digit) is twice the value of 3 (the ten's digit), making the first condition true. Twice of 36 is 72, which is 9 more than the reversed number, which is 63.

write an equation in slope -intercept form for the line that passes through the points (8,-3) and has a slope of 3 over 2

Answers

Answer:

y = 3/2x -15

Step-by-step explanation:

y = mx + b this is the slope intercept form.  We need the slope and the y intercept.  The slope (m) has been given (3/2), we just need the b (y-intercept).  We will use the point given to find b.  The point given is (8,-3).  8 is the x term and -3 is the y term.  We will plug them into y = 3/2x + b to solve for b.

y = mx + b

y = (3/2) x + b   The slope was given

-3 = (3/2)(8) + b  Plugged in the given x and y from the point (8, -3)

-3 = 12 + b  I can make 8 into a fraction by putting a 1 under it.

                   (3/2)(8/1).  I can cross cancel to make the number easier to

                    2 and 8 both share the factor 2 so I will divide both numbers

                   by 2 and I will now have (3/1)(4/1).  3x4 is 12 and 1x1 is 1, so

                   12/1 is  the same as 12.

-15 = b      Subtract both sides by 12.  -3-12 is -15

Now that I have the m and the b, I can write the equation in the slope intercept form.  y =  3/2 + (-15) or y = 3/2 -15    

What is the unit price of a Mt. Dew if a six packs costs $2.70

Answers

Answer:

270 uits

Step-by-step explanation:

Find the measure of x

Answers

Answer:

113°

Step-by-step explanation:

Angles that form a linear pair are supplementary, and thus add to 180°.

The average heart rate of an adult human is 72 beats per minute. For an adult elephant, the average heart rate is 35 beats per minute.

A. Whose heart beats more times in one hour

B. Whose heart makes 1,000,000 beats in less time

Answers

The answer for both questions are adult humans

10 - 13x + 2 in words

Answers

The simplification of the equation above in words is = Twelve minus thirteen X.

Simplification of an equation

In mathematics, coefficients are those numbers that multiplies a variable. For example in 13x, the coefficient is 13 which multiplies a variable known as X.

The equation given is,

10 - 13x + 2

Arrange the digits without a coefficient together.

That is ,

10+2 -13x

12 - 13x

Therefore the simplification of the given equation in words is Twelve minus thirteen X.

Learn more about coefficients here:

https://brainly.com/question/1760174

#SPJ1

Given angle ABC shown on the coordinate plane below.

Draw angle A”B””C” = Ro 90(T<-4,3>(ABC))

Answers

The attached image shows the image of A"B"C" after the transformation

What is the transformation of the triangle about?

The transformation rule states:

A"B"C" = Ro90° (T(-4,3)(ABC))

This implies that one need to rotate the triangle in a 90⁰ clockwise direction, and then one need to translate the triangle.

Using the image shown, the coordinates of ABC are;

A = (-1, 2)

B = (1, 4)

C = (3, -1)

The 90⁰ rule clockwise rotation  will be:

(x,y) -- (y,-x)

So, when translated, it will be:

A' = (2, 1)

B' = (4, -1)

C' = (-1, -3)

Then the translation of the triangle using T(-4,3):

(x, y) -  (x - 4, y + 3)

So, there is:

A'' = (-2, 4)

B'' = (0, 2)

C'' = (-5, 0)

Learn more about transformation from:

https://brainly.com/question/28108536

#SPJ1

Asap I need a answer please

Answers

Answer:

-4

Step-by-step explanation:

in order to get from -6 to 24, you need to multiply x by -4 since -6* -4 is 24

Find a12 given the geometric sequence 4, 8, 16, 32, ...
A. 32,768
OB. 8192
OC. 16,384
D. 4096
Reset Selection

Answers

The twelfth element of the geometric sequence is equal to 4,096. (Correct choice: D)

How to find a determined element of a geometric sequence by exponential formulae

Sequences are series of elements generated according to at least one condition, usually equations. geometric sequences are generated according to a exponential formulas, whose form and characteristics are described below:

f(n) = a · bⁿ ⁻ ¹    (1)

Where:

a - First element of geometric sequenceb - Common ratio of the geometric sequencen - Element index within the geometric sequence

If we know that a = 4, b = 2 and n = 12, then the twelfth element of the geometric sequence from the statement is:

f(12) = 4 · 2¹² ⁻ ¹

f(12) = 4 · 2¹¹

f(12) = 4 · 2,048

f(12) = 4,096

The twelfth element of the geometric sequence is equal to 4,096. (Correct choice: D)

To learn more on geometric sequences: https://brainly.com/question/4617980

#SPJ1

slove each problem include a diagram. a. A 6m long wheelchair ramp makes an angle of 15 with the ground. How high abouve the ground does the top end of the ramp reach. b.two treees are 120m apart. From the point halfway between them, then angle of the elevation to the top of the trees is 36 and 52. how much taller is one tree than the other.

Answers

The height of the ramp above the ground is 1.55 meters.

The difference between the height of the tree is 33.20 meters

How to find sides of a right triangle?

The situation forms a right angle triangle.

Therefore, the height the ramp above the ground can be calculated as follows:

Using Pythagoras theorem,

sin 15  = opposite / hypotenuse

sin 15 = x / 6

cross multiply

x = 6 sin 15

x = 6 × 0.2588190451

x = 1.55291427062

x = 1.55

Therefore, the height of the ramp above the ground is 1.55 meters

tan 36  = opposite / adjacent

tan 36 = x / 60

x = 60 tan 36

x = 43.5925516803

x = 43.60 meters

tan 52 = y / 60

60 tan 52 = y

y = 76.7964979316

y = 76.79 meters

The difference between the height of the tree = 76.79 - 43.60 = 33.20 meters

learn more on right triangle here: https://brainly.com/question/8036473

#SPJ1

The vertices of a quadrilateral are A(-3,-1), B(1,5), C(5,5), and D(5,-1). Select the statement that represents this quadrilateral. A. ABCD is a rectangle because it has exactly one pair of right angles. B. ABCD is a trapezoid because it has at least one pair of parallel sides. C. ABCD is a square because it has all equal sides. D. ABCD is a parallelogram because it has two pairs of parallel sides.

Answers

Answer:

B. ABCD is a trapezoid because it has at least

one pair of parallel sides.  

Step-by-step explanation:

the two points A(-3,-1) and D(5,-1) have the same y-coordinates

Then

The line AD is parallel to the x-axis

On the other hand,

the two points B(1,5) and C(5,5) have the same y-coordinates

Then

The line BC is parallel to the x-axis.

We obtain :

• AD is parallel to the x-axis

• BC is parallel to the x-axis.

Therefore

AD // BC

Conclusion:

ABCD is a trapezoid because it has at least

one pair of parallel sides. 

Answer:

b

Step-by-step explanation:

plato

Company A manufactures and sells gidgets. The owners have determined that the company has the monthly revenue and cost functions shown, such that x represents the number of gidgets sold.

R(x) = 16x
C(x) = 12x + 1,424

At what number of gidgets sold will the company break-even (the point where revenue equals cost)?

Answers

Considering the given equations for revenue and cost, the company will break-even when 356 widgets are sold.

What is the break-even point?

The break-even point is the value of x for which:

R(x) = C(x).

In this problem, the functions are:

R(x) = 16x.C(x) = 12x + 1424.

Hence:

R(x) = C(x)

16x = 12x + 1424

4x = 1424

x = 1424/4

x = 356

The company will break-even when 356 widgets are sold.

More can be learned about the break-even point of a company at https://brainly.com/question/9212451

#SPJ1

I need help with this geometry question asap!

Answers

Answer:  True

Explanation:  Parallel lines are two lines that lie on the same plane and they never intersect.  

Q.3. Set up the equations and solve them to find the unknown numbers in the cases given below:
1) If you add 6 to five times a number, it gives 46.
2) Two-third of a number minus 5 gives 11.
3) If you take onethird of a number and add 4 to it, gives 45.
4) When person X subtracts 12 from thrice of a number, it gives 18.
5) When Jenny subtracts twice the number of pens, she has from 40, she gets 16.
6) Virat guesses a number. If he adds 18 to that number and then divides the sum by 6, he gets answer 7.
7) Ami guesses anumber. If she subtracts 8 from two third of a number, she gets 6

I want Answer quickly ​

Answers

The following expressions set up as an equation to solve for the unknown numbers in the cases given below:

Algebraic equation

let

The unknown number = x

5x + 6 = 46

5x = 46 - 6

5x = 40

x = 40/5

x = 8

2/3x - 5 = 11

2/3x = 11 + 5

2/3x = 16

x = 16 ÷ 2/3

x = 16 × 3/2

x = 48/2

x = 24

1/3x + 4 = 45

1/3x = 45 - 4

1/3x = 41

x = 41 ÷ 1/3

x = 41 × 3/1

x = 123

3x - 12 = 18

3x = 18 + 12

3x = 30

x = 30/3

x = 10

40 - 2x = 16

-2x = 16 - 40

-2x = -24

x = -24/-2

x = 12

(x + 18) / 6 = 7

(x + 18) = 7 × 6

x + 18 = 42

x = 42 - 18

x = 24

2/3x - 8 = 6

2/3x = 6 + 8

2/3x = 14

x = 14 ÷ 2/3

x = 14 × 3/2

= 42/2

x = 21

Learn more about algebraic equation:

https://brainly.com/question/4344214

#SPJ1

The length of the major axis of the ellipse below is 14, and the length of the
red line segment is 3. How long is the blue line segment?

Answers

Answer:

11

Step-by-step explanation:

The focal length is 14, and thus the sum of the lengths of the blue and red segments is also 14.

Allen's hummingbird (Selasphorus sasin) has been studied by zoologist Bill Alther.† Suppose a small group of 17 Allen's hummingbirds has been under study in Arizona. The average weight for these birds is x = 3.15 grams. Based on previous studies, we can assume that the weights of Allen's hummingbirds have a normal distribution, with = 0.40 gram.
(a)
Find an 80% confidence interval for the average weights of Allen's hummingbirds in the study region. What is the margin of error? (Round your answers to two decimal places.)
lower limit
upper limit
margin of error
(b)
What conditions are necessary for your calculations? (Select all that apply.)
normal distribution of weights
is unknown
uniform distribution of weights
is known
n is large
Correct: Your answer is correct.

(c)
Interpret your results in the context of this problem.
The probability that this interval contains the true average weight of Allen's hummingbirds is 0.80.
We are 20% confident that the true average weight of Allen's hummingbirds falls within this interval.
We are 80% confident that the true average weight of Allen's hummingbirds falls within this interval.
The probability that this interval contains the true average weight of Allen's hummingbirds is 0.20.
Correct: Your answer is correct.
(d)
Find the sample size necessary for an 80% confidence level with a maximal margin of error E = 0.13 for the mean weights of the hummingbirds. (Round up to the nearest whole number.)

Incorrect: Your answer is incorrect.
hummingbirds

Answers

a)

E =0.124Lower limit =3.026Upper limit =  3.274

b) Conditions:[tex]\sigma[/tex] is understood to have a regular distribution of weights

c) Interpretation: 80 percent of the time, this range will include the actual average weight of Allen's hummingbirds, hence its reliability may be counted on

d) sample size [tex]n \approx 27[/tex].

What conditions are necessary for your calculations?

Generally,
Here n = 17

T= 3.15

[tex]\sigma=0.40[/tex]

confidence level =c = 0.80

Here we will usethe  z critical value.

z critical value for (1+c) /2 = (1+0.80)/2

z critical value for (1+c) /2= 0.9

Zc = 1.28

Critical value = 1.28

a) Margin of error (E) :

[tex]E = Zc*\frac{\sigma}{\sqrt{n}}\\\\\E = 1.28*\frac{0.40}{\sqrt{17}}[/tex]

E =0.124

Margin of error = E =0.124

Lower limit = x-E

L= 3.15 - 0.124

L= 3.026

Lower limit =3.026

Upper limit = x + E

U= 3.15 + 0.124

U= 3.274

Upper limit =  3.274

The following is a confidence interval with a value of 80 percent for the mean weights of Allen's hummingbirds in the area under study: (3.02,3.28)

b) Conditions:[tex]\sigma[/tex] is understood to have a regular distribution of weights

c) Interpretation: 80 percent of the time, this range will include the actual average weight of Allen's hummingbirds, hence its reliability may be counted on.

d) When is already known, the following equation may be used to determine the appropriate number of samples, n:

[tex]n=(\frac{z_c*\sigma }{E})^2[/tex]

Margin of error = 0.13

Sample size (n):

[tex]n= (\frac{z_c*\sigma }{E})^2[/tex]

[tex]n=(\frac{ 1.28*0.36}{0.09})^2[/tex]

n = 26.21

[tex]n \approx 27[/tex]

Read more about probability

https://brainly.com/question/795909

#SPJ1

What would you have to calculate to prove the figure below is a SQUARE?
O None of these choices are correct.
O use the distance formula to show that the opposite sides are supplementary
the sides all have the same slopes
O diagonals are ½ the length of the midpoint
slopes are perpendicular where the sides meet

Answers

The proof that the diagram is a square is that; D: slopes are perpendicular where the sides meet

How to Prove a quadrilateral is a Square?

We know that a square is a quadrilateral that has 4 sides. Now, the 4 sides of a square are all equal and at right angles.

Since all sides of the square are equal, then it means that 2 consecutive sides must be perpendicular to each other. Thus, this means that the slope of 2 consecutive sides must be perpendicular to each other.

Read more about Square proof at; https://brainly.com/question/4471785

#SPJ1

Other Questions
Dr. Berry, who employs the ecological approach to human development, is studying the relationship between the quality of parent-child interactions in the home and the success of children in school. Dr. Berry is studying development at the ________ level of environmental influence. Has anyone collected the rubbish yet ?change it to passive voice thanks A 32-year-old sees her obstetrician about a lump in the lower outer quadrant of the right breast. Her mother and aunt both have a history of breast cancer. What diagnosis code(s) should be reported ( 1 ) 0.006 m (cm) =( 2 ) 17, 605 mm (km) = ( 3 ) 1,001,00 dm (km) = Micheal measured the distance around the circler swimming pool as apppoximly 108 feet which of the following represents the approximates radius of the circular swimming pool The scatterplot to the left shows the cost, CCC, in thousands of dollars, and living space, xxx, in square feet (\text{ft}^2)(ft 2 )left parenthesis, start text, f, t, end text, squared, right parenthesis for several houses in a certain neighborhood. According to the data, which of the following best approximates the cost for an additional square foot of living space for homes in this neighborhood?Choose 1 answer:\$80$80dollar sign, 80\$300$300dollar sign, 300\$1{,}000$1,000dollar sign, 1, comma, 000\$13{,}000$13,000dollar sign, 13, comma, 000 3x-6=15 what is the value of x? ________ are the most important monetary policy tool because they are the primary determinant of changes in the ______, the main source of fluctuations in the money supply. complete the dialogues When an event takes place, the control responsible for the event creates an event? The composition of a given pure compound is always no matter what the source of the compound? 5. Where is the graph increasing? Refer to the figure above. Assume that the graphs in this figure represent the demand and supply curves for automobiles. Which panel best describes what happens in this market when the price of steel falls 1. Which of the following results in the difference of two squares?O (3r-5y) (3x - 5y)O (5x+3y) (5x-3y)O 4r(3r-9)O (2r-3y)2 His belligerent disposition made him completely unlikable.a. meanb. sweetc. deviousd. quarrelsome Anger is the toxic core of: Part E:Write the equation of the graphed function f(x), where a is the leading coefficient. Use the factors found in part D. Express the function as the product of its leading coefficient and the expanded form of the equation in standard form. Assume the union freeman works with pays newer employees less than ones who have been with the organization for a longer time. what can be the basis for this? I can find the perimeter and area of the rectangle Which is the graph of f(x) = }(4)*2